Q12

User avatar
 
LSAT-Chang
Thanks Received: 38
Atticus Finch
Atticus Finch
 
Posts: 479
Joined: June 03rd, 2011
 
 
trophy
Most Thankful
trophy
First Responder
 

Q12

by LSAT-Chang Tue Sep 20, 2011 12:48 am

Could someone please help me understand why C is correct? I thought A was correct because we get evidence from passage A that furbearer populations reduced whereas passage B states the opposite. Also, why is B wrong? What would the author of passage B have to assume about A to be correct?
User avatar
 
maryadkins
Thanks Received: 641
Atticus Finch
Atticus Finch
 
Posts: 1261
Joined: March 23rd, 2011
 
 
 

Re: Q12

by maryadkins Thu Sep 22, 2011 2:09 pm

Passage A is about controlling the purple loosestrife (what a name) and the challenges in doing so. It does not consider the possibility that purple loosestrife shouldn't be controlled, or that trying to control it is either unnecessary or motivated by unjustifiable reasons. Passage B discusses the point of view of Passage A--that the loosestrife should be controlled--and why it is unfounded. (C) nails this.

(A) is incorrect but tempting. If we look closely, the evidence in both passages doesn't actually conflict. Passage A says waterfowl is threatened, and Passage B says no birds BUT the canvasback (in other words, yes, still birds). Passage A mentions declines in furbearer productivity, but Passage B says none of certain examples of furbearing animals (mink) can be considered threatened. These specific examples are not mentioned in Passage A.

(B) is incorrect because Passage B doesn't assume what Passage A argues for--it addresses it explicitly and directly.

(D) is incorrect because Passage B doesn't advocate a policy. It argues against the "war on purple loosestrife."

(E) is incorrect because Passage A doesn't even address the claims in Passage B.
 
gmatalongthewatchtower
Thanks Received: 1
Jackie Chiles
Jackie Chiles
 
Posts: 47
Joined: November 22nd, 2011
 
 
 

Re: Q12

by gmatalongthewatchtower Fri Apr 13, 2012 3:15 pm

maryadkins Wrote:Passage A is about controlling the purple loosestrife (what a name) and the challenges in doing so. It does not consider the possibility that purple loosestrife shouldn't be controlled, or that trying to control it is either unnecessary or motivated by unjustifiable reasons. Passage B discusses the point of view of Passage A--that the loosestrife should be controlled--and why it is unfounded. (C) nails this.

(A) is incorrect but tempting. If we look closely, the evidence in both passages doesn't actually conflict. Passage A says waterfowl is threatened, and Passage B says no birds BUT the canvasback (in other words, yes, still birds). Passage A mentions declines in furbearer productivity, but Passage B says none of certain examples of furbearing animals (mink) can be considered threatened. These specific examples are not mentioned in Passage A.

(B) is incorrect because Passage B doesn't assume what Passage A argues for--it addresses it explicitly and directly.

(D) is incorrect because Passage B doesn't advocate a policy. It argues against the "war on purple loosestrife."

(E) is incorrect because Passage A doesn't even address the claims in Passage B.



I am still not clear why B is incorrect. As you stated, A says that controls are required. B assumes that controls are required but the underlying reasons are different - the economics vs. saving the plants and animals.

Any thoughts?

Thanks
Voodoo
User avatar
 
maryadkins
Thanks Received: 641
Atticus Finch
Atticus Finch
 
Posts: 1261
Joined: March 23rd, 2011
 
This post thanked 1 time.
 
 

Re: Q12

by maryadkins Sun Apr 15, 2012 1:19 pm

gmatalongthewatchtower Wrote:B assumes that controls are required but the underlying reasons are different - the economics vs. saving the plants and animals.


Careful. Does Passage B assume control is required? If anything, Passage B implies that efforts to control are motivated by human desires to control nature which is a bad thing (line 50). Re-read the lines about the economics of the situation. There is no assumption here that the plant needs to be controlled. The discussion is just about what's motivating those trying to control it.
 
gmatalongthewatchtower
Thanks Received: 1
Jackie Chiles
Jackie Chiles
 
Posts: 47
Joined: November 22nd, 2011
 
 
 

Re: Q12

by gmatalongthewatchtower Wed Apr 18, 2012 11:36 am

Thanks for your response. I have a quick question :

Passage states that "....Consequently, the
scientific effort to liberate nature from purple
loosestrife has failed to decouple itself from its
philosophical origin as an instrument to control nature
(50) to the satisfaction of human desires."

Doesn't it mean that Passage A talks about control the population of PL but the community ends up satisfying the needs of humans?

Can you please help me? I am little bit confused.

Thanks
Voodoo child
User avatar
 
maryadkins
Thanks Received: 641
Atticus Finch
Atticus Finch
 
Posts: 1261
Joined: March 23rd, 2011
 
 
 

Re: Q12

by maryadkins Thu Apr 26, 2012 3:27 pm

gmatalongthewatchtower Wrote:Doesn't it mean that Passage A talks about control the population of PL but the community ends up satisfying the needs of humans?


I believe the text you quoted means that the attempt to control is in order to satisfy human desires. I'm not sure if they ARE satisfied or not. But regardless, the question is whether (B) assumes control is necessary (or a good thing, or desirable), which it definitely doesn't. It's tone is critical of controlling PL.
 
tzyc
Thanks Received: 0
Atticus Finch
Atticus Finch
 
Posts: 323
Joined: May 27th, 2012
 
 
trophy
Most Thankful
 

Re: Q12

by tzyc Sun Apr 07, 2013 12:01 am

Does "awareness" just refer to passage B touched what passage A says or mentioned the same topic?

Thanks!
 
timsportschuetz
Thanks Received: 46
Elle Woods
Elle Woods
 
Posts: 95
Joined: June 30th, 2013
 
 
trophy
First Responder
 

Re: Q12

by timsportschuetz Fri Nov 29, 2013 9:48 pm

Firstly, if the answer choice states "presents evidence that directly counters claims...", you must interpret this at face value! So, answer (A) states that passage A "directly counters" claims made in passage B. Previous posters have discussed this particular answer choice by carefully analysing the presented evidence in each, however, the key to solving this problem is the words "directly counters"! Does passage A "directly counter" claims made in passage B? No, it does not do so anywhere. Only passage B is explicitly aware of some of the evidence of passage A - not the other way around!